Difference between revisions of "1989 AIME Problems/Problem 1"

m (fmt, box)
Line 3: Line 3:
  
 
== Solution ==
 
== Solution ==
Let's call our four consecutive integers <math>(n-1), n, (n+1), (n+2)</math>.  Notice that <math>(n-1)(n)(n+1)(n+2)+1=(n^2+n)^2-2(n^2+n)+1 \Rightarrow (n^2+n-1)^2</math>.  Thus, <math>\sqrt{(31)(30)(29)(28)+1} = (29^2+29-1) = 869</math>
+
Let's call our four [[consecutive]] integers <math>(n-1), n, (n+1), (n+2)</math>.  Notice that <math>\displaystyle (n-1)(n)(n+1)(n+2)+1=(n^2+n)^2-2(n^2+n)+1 \Rightarrow (n^2+n-1)^2</math>.  Thus, <math>\sqrt{(31)(30)(29)(28)+1} = (29^2+29-1) = 869</math>
  
 
== See also ==
 
== See also ==
* [[1989 AIME Problems/Problem 2|Next Problem]]
+
{{AIME box|year=1989|before=First Question|num-a=2}}
* [[1989 AIME Problems]]
 

Revision as of 20:27, 26 February 2007

Problem

Compute $\sqrt{(31)(30)(29)(28)+1}$.

Solution

Let's call our four consecutive integers $(n-1), n, (n+1), (n+2)$. Notice that $\displaystyle (n-1)(n)(n+1)(n+2)+1=(n^2+n)^2-2(n^2+n)+1 \Rightarrow (n^2+n-1)^2$. Thus, $\sqrt{(31)(30)(29)(28)+1} = (29^2+29-1) = 869$

See also

1989 AIME (ProblemsAnswer KeyResources)
Preceded by
First Question
Followed by
Problem 2
1 2 3 4 5 6 7 8 9 10 11 12 13 14 15
All AIME Problems and Solutions